Estimar el nro. De términos que deberían incluirse en la serie para aproximar ln(1.5) con un margen de error no mayor que 10^-10

Tema: Series de Taylos. Error.

Estimar el nro. De términos que deberían incluirse en la serie para aproximar ln(1.5) con un margen de error no mayor que 10^-10

Respuesta
2

·

·

¡Hola Patricio!

Primero calculamos la serie

$$\begin{align}&f(x)=ln(1+x)\\&\\&f'(x)=\frac{1}{1+x}=(1+x)^{-1}\\&\\&f''(x)=-(1+x)^{-2}\\&\\&f'''(x)=2(1+x)^{-3}\\&\\&f^{(4)}(x)= -6(1+x)^{-4}\\&\\&f^{(n)}(x)= (-1)^{n+1}(n-1)!(1+x)^{-n}\\&\\&\text{Estas derivadas para x=0 son}\\&\\&f^{(n)}(0)= (-1)^{n+1}(n-1)!\\&\\&\text{Y el error para n términos en x=0.5 es}\\&\\&E=\frac{f^{(n+1)}(0)}{(n+1)!}x^{n+1}\quad\text{con }0\le x\le0.5\\&\\&\text{El valor máximo de E es para }x=0.5\\&\\&\left|\frac{(-1)^{n+2}(n)!}{(n+1)!}·0.5^{n+1}  \right|\le 10^{-10}\\&\\&\left| \frac{0.5^{n+1}}{n+1}\right|\le 10^{-10}\\&\\&\frac{0.5^{n+1}}{n+1}\le10^{-10}\\&\\&\text{Esto no se puede resolver algebraicamente de golpe}\\&\text{pero se pueden hacer cosas}\\&\\&\frac{0.5^{n+1}}{n+1}\le 0.5^{n+1}\le10^{-10}\\&\\&\text{Sacamos logaritmos}\\&\\&(n+1)ln(0.5)\le -10ln(10)\\&\\&\text{Como }ln(0.5) \text{ es negativo}\\&\\&n+1\ge \frac{-10ln(10)}{ln(0.5)}\\&\\&n \ge \frac{-10ln(10)}{ln(0.5)}-1\\&\\&n\ge 32.22\\&\\&\text{Ahora viene lo bueno. Hemos sido muy generosos,}\\&\text{seguro que n será menor que eso, usamos la cota}\\&\text{obtenida para afinar más}\\&\\&\frac{0.5^{n+1}}{n+1}\le \frac{0.5^{n+1}}{33.22}\le10^{-10}\\&\\&0.5^{n+1}\le 33.22·10^{-10}\\&\\&(n+1) ln(0.5)\le ln(33.22·10^{-10})\\&\\&n+1\ge \frac{ln(33.22·10^{-10})}{ln (0.5)}=28.165\\&\\&n\ge27.16\end{align}$$

Y como n debe ser entero tomaremos n=28

:

:

Muchas gracias por la respuesta Valero! Quedó claro, y me sirvió la forma en la que resolviste la n algebraicamente ya que yo al final fui probando valores hasta que se cumplió. Como había comentado más abajo, la fórmula del residuo esta definida como:

$$\begin{align}&f^(N+1)(ψ)/(N+1)!*x^(N+1)\end{align}$$

Luego veo que tomaste ese Omega igual a 0, como yo tambien hice, lo cual simplifica mucho los calculos. Ese omega se supone que esta entre por y a. La cuestión es, si soy libre de elegir el Omega que me convenga en ese intervalo, o no, es decir, se supone que con cualquier Omega llegaría a un resultado válido, solo que cambiaría la precisión del error, ¿correcto? En otras palabras, por que tu eliges como Omega 0 y no 0.5 por ejemplo. Muchas gracias! Espero puedas aclararme esa duda.

Nuevamente la pregunta, había errores de tipeo:

Muchas gracias por la respuesta Valero! Quedó claro, y me sirvió la forma en la que resolviste la n algebraicamente ya que yo al final fui probando valores hasta que se cumplió. Como había comentado más abajo, la fórmula del residuo esta definida como:

f(N+1)(ψ)/(N+1)!∗x^(N+1)

Luego veo que tomaste ese Omega igual a 0, como yo también hice, lo cual simplifica mucho los cálculos. Ese omega se supone que esta entre x y a. La cuestión es, si soy libre de elegir el Omega que me convenga en ese intervalo, o no, es decir, se supone que con cualquier Omega llegaría a un resultado válido, solo que cambiaría la precisión del error, ¿correcto? En otras palabras, por que tu eliges como Omega 0 y no 0.5 por ejemplo. Muchas gracias! Espero puedas aclararme esa duda.

Me traicionó la memoria, debía haber mirado la fórmula de Taylor antes de contestar que hacía años que no la veía. No me acordaba que en el término del error lo que variaba entre a y x (siendo a el centro y x el punto donde calculamos) era el punto donde se tomaba la derivada n+1. Bueno, pues entonces no lo hice bien, voy a repetirlo.

$$\begin{align}&f(x)=ln(1+x)\\&\\&f'(x)=\frac{1}{1+x}=(1+x)^{-1}\\&\\&f''(x)=-(1+x)^{-2}\\&\\&f'''(x)=2(1+x)^{-3}\\&\\&f^{(4)}(x)= -6(1+x)^{-4}\\&\\&f^{(n)}(x)= (-1)^{n+1}(n-1)!(1+x)^{-n}\\&\\&\text{Estas derivadas para x=0 son}\\&\\&f^{(n)}(0)= (-1)^{n+1}(n-1)!\\&\\&\text{Y el error para n términos en x=0.5 es}\\&\\&E=\frac{f^{(n+1)}(\xi)}{(n+1)!}0.5^{n+1}\quad\text{con }0\le \xi\le0.5\\&\\&\text{Como } f^{(n+1)}(\xi)=\frac{(-1)^{n+2}(n)!}{(1+\xi)^{n+1}}\\&\\&\text{El máximo de E en valor absoluto se da con }\xi=0\\&\\&\text{por eso tendremos }\\&\\&|E|\le \left|\frac{(-1)^{n+2}(n)!}{(n+1)!}·0.5^{n+1}  \right|\le 10^{-10}\\&\\&\left| \frac{0.5^{n+1}}{n+1}\right|\le 10^{-10}\\&\\&\frac{0.5^{n+1}}{n+1}\le10^{-10}\\&\\&\text{Esto no se puede resolver algebraicamente de golpe}\\&\text{pero se pueden hacer cosas}\\&\\&\frac{0.5^{n+1}}{n+1}\le 0.5^{n+1}\le10^{-10}\\&\\&\text{Sacamos logaritmos}\\&\\&(n+1)ln(0.5)\le -10ln(10)\\&\\&\text{Como }ln(0.5) \text{ es negativo}\\&\\&n+1\ge \frac{-10ln(10)}{ln(0.5)}\\&\\&n \ge \frac{-10ln(10)}{ln(0.5)}-1\\&\\&n\ge 32.22\\&\\&\text{Ahora viene lo bueno. Hemos sido muy generosos,}\\&\text{seguro que n será menor que eso, usamos la cota}\\&\text{obtenida para afinar más}\\&\\&\frac{0.5^{n+1}}{n+1}\le \frac{0.5^{n+1}}{33.22}\le10^{-10}\\&\\&0.5^{n+1}\le 33.22·10^{-10}\\&\\&(n+1) ln(0.5)\le ln(33.22·10^{-10})\\&\\&n+1\ge \frac{ln(33.22·10^{-10})}{ln (0.5)}=28.165\\&\\&n\ge27.16\end{align}$$

Ahora ya está perdona por no haberlo hecho bien antes.  El valor de xi tienes que tomarlo de modo que se maximice el valor absoluto del error, en este caso era con xi=0 pero podria haber sido cualquier otro dentro del intervalo (0, 0.5)

Perfecto! Ahora si, esa era mi duda y estuve leyendo un poco en libros sobre como elegir bien el xi, y si, es siempre el valor que más me convenga, para hacer que el error sea menor. En este caso, además eligiendo xi igual a 0 se llega al resultado exacto, pero como dices, xi puede ser cualquier valor en ese intervalo, el tema es que me convenga.

Muchas gracias!

Cuidado, no es el que haga menor el error sino el que lo haga mayor.

1 respuesta más de otro experto

Respuesta
1

Veamos algunas derivadas para ver si podemos estimar la forma general

f(

$$\begin{align}&f(x) = ln(1+x)\\&f'(x) = (1+x)^{-1}\\&f''(x) = - (1+x)^{-2}\\&f'''(x) = 2 (1+x)^{-3}\\&f^{iv}(x)= -6(1+x)^{-4}\\&f^{v}(x)= 24(1+x)^{-5}\\&\text{Por lo que el término general de la derivada enésima parece ser}\\&f^{n}(x)= (-1)^{n+1}(n-1)! (1+x)^{-n}\\&\text{Y el Polinomio de Taylor centrada en 0, sería}\\&P(x) = \sum_{n=0}^{N} \frac{f^{(n)}(1)}{n!}\cdot x^n + \frac{f^{(N+1)}(\psi)}{(N+1)!}x^{N+1}  \text{.........(Con }-1 < \psi < 1)\\&\text{Donde el último término define el error}\\&\text{Como nos piden que este error sea no mayor que }10^{-10} \text{ lo que dicen es}\\&\bigg|\frac{f^{(N+1)}(\psi)}{(N+1)!}x^{N+1} \bigg|\le 10^{-10}\\&\bigg|\frac{f^{(N+1)}(\psi)}{(N+1)!}x^{N+1}\bigg| = \bigg|(-1)^{N+2}(N-1+1)! (1+\psi)^{-N+1}\cdot \frac{1}{(N+1)!}\cdot x^{N+1}\bigg|=\\&\bigg|(-1)^{N+2}N! (1+\psi)^{-N+1}\cdot \frac{1}{(N+1)!}\cdot x^{N+1}\bigg|=\\&\bigg|(-1)^{N+2}\bigg|\cdot  \bigg| (1+\psi)^{-N+1}\bigg| \cdot \bigg|\frac{1}{(N+1)}\bigg| \cdot \bigg| x^{N+1}\bigg|=\\&\bigg| (1+\psi)^{-N+1}\bigg| \cdot \bigg|\frac{1}{(N+1)}\bigg| \cdot \bigg| x^{N+1}\bigg|\\&\text{Como 0 < x < 1 y -1 <} \psi <1\\&\bigg| (1+\psi)^{-N+1}\bigg| \cdot \bigg|\frac{1}{(N+1)}\bigg| \cdot \bigg| x^{N+1}\bigg|\le \bigg|\frac{1}{(N+1)}\bigg|= \frac{1}{N+1}\\&\text{ y queremos que}\\&\frac{1}{N+1} \le 10^{-10}\\&10^{10} \le N+1\\&N \ge 10^{10}-1\end{align}$$
$$\begin{align}&\\&\end{align}$$

En primer lugar, muchísimas gracias por tu esfuerzo y responderme.Afortunadamente he logrado resolver el ejercicio por mi cuenta, y te voy a hacer solo algunas correciones, el resto está muy bien. Al final, recuerda que la x está dada, si tomamos x=0.5, por lo que el término x^N+1 debe incluirse en la cota, seria (1/2)^N+1*u(1/N+1). Luego recuerda que Omega tambien lo podemos sacar algebraicamente, ya que está entre 1 y -1, por lo tanto lo hacemos igual a 0 y el término se va. Luego resolviendo lo que nos quedó, nos queda que N es igual a 28, lo cual lo verifiqué con un software matemático y efectivamente, si aplicamos 28 iteraciones a la serie se obtienen 10 digitos de presición. Aclaro esto para los proximos que lo lean, de todas formas el trabajo que hiciste esta bien.

Por favor Gustavo, te agradecería enormemente si pudieras ayudarme con este ejercicio:

Desarrolle la fucion raiz(x) en serie de potencias centrada en x=1 y verifique que utilizando la aproximacion lineal de dicha.. 

Gracias nuevamente

Ojo con Omega, porque si bien está entre -1 y 1, no puedes asegurar que sea 0, así que ese término no lo podés eliminar.

Me queda esa duda. No termino de entender, si ese omega es simplemente una cota, es decir, al ser una cota, la cual el teorema nos asegura que está entre x y a, entonces la variación del omega que elijamos, será más grande o más chico el error, pero la igualdad se sigue cumpliendo. Por eso elijo omega como 0 por conveniencia algebraica, pero podría haber elegido otro valor. Si no es así espero me puedas aclarar mis ideas. Gracias 

Añade tu respuesta

Haz clic para o

Más respuestas relacionadas